LSAT and Law School Admissions Forum

Get expert LSAT preparation and law school admissions advice from PowerScore Test Preparation.

 lanereuden
  • Posts: 147
  • Joined: May 30, 2019
|
#66731
Thinking A here is correct because genetic predisposition should not be limited to one gene (I.e. fails to consider that complex genome exists that would be sufficient to assert gene predispositions)

Yes?
 lanereuden
  • Posts: 147
  • Joined: May 30, 2019
|
#67214
Thinking A here is correct because genetic predisposition should not be limited to one gene (I.e. fails to consider that complex genome exists that would be sufficient to assert gene predispositions)

Yes?

Is this the right page for this above question?
 adkirk88
  • Posts: 9
  • Joined: Aug 08, 2019
|
#67228
I have a question about this as well.

How are we to know Chang presupposes only one possibility exists? Is it because in Wirth's argument it is stated that "nearly all researchers now agree that there is no 'manic-depression gene', Chang claims to agree with Wirth's evidence, but then contradicts it in their conclusion?

I thought B was correct because Chang's conclusion depended on another piece of evidence that contradicts the argument Wirth makes that Chang affirms; leading Chang to contradict him/herself. Where am I off?
 Zach Foreman
PowerScore Staff
  • PowerScore Staff
  • Posts: 91
  • Joined: Apr 11, 2019
|
#67258
lane and adkirk,
This is a difficult question. We have two arguments, Wirth's and Chang's and it is important to keep the two distinct.
First, we need to understand each argument and then the question stem.

Wirth's argument boils down to p1: none of the many attempts to find a m-d gene has succeeded p2: nearly all researchers agree that there is no (single) m-d gene. C: No one is genetically predisposed to m-d

Chang's argument is there are several genes involved that interact and from that is the disposition to m-d. This means that there is, in fact, a genetic predisposition.

First, we note that Chang is explicitly accepting the two premises. He is rejecting the conclusion, but he is not explicit as to why. The best place to prephrase this flaw in Wirth's reasoning would be while reading the stimulus. In this case, it is begging to be found because of Chang's disagreement. So what is the flaw?

Well, just because m-d doesn't result from a single gene doesn't mean there can be NO genetic predisposition because there is a missing possibility, namely multiple genes causing a predisposition to manic depression.

So, now we understand the premise, we turn to the question stem. It is a second order question because it is not simply asking for one argument or the other but for Chang's criticism of Wirth's argument. We can use our prephrase: Chang thinks Wirth's argument is flawed because he ignores the possibility that a genetic predisposition can be caused by multiple genes.
That prephrase gets us to A. The "it" in the answer choice is confusing, but "it" refers to Wirth's argument. So, if we were to flesh out the answer (always a good thing to do for these abstract answers), we would write "Wirth's argument presupposes only one possibility (genetic dispositions are caused by single genes only) where more than one (genetic predispositions can be caused by single genes OR by multiple genes) exists."
So, adkirk, you are off because you are primarily analyzing potential flaws in Chang's argument (you say "how do we know Chang presupposes..." and "leading Chang to contradict...") We are not supposed to look for flaws in Chang's argument. On the contrary, we are to "think like Chang" and find flaws in Wirth's argument, i.e. "accurately express Chang's criticism". The first "It" in each answer choice should be "Wirth's argument" NOT "Chang's argument".
 adkirk88
  • Posts: 9
  • Joined: Aug 08, 2019
|
#67333
Thanks for the help Zach that makes a lot of sense! Honestly I think I was critiquing Chang rather than Wirth and the explanation of "IT" in the answer choices confirmed that. Thanks for the thorough explanation!
User avatar
 ashpine17
  • Posts: 321
  • Joined: Apr 06, 2021
|
#88512
But why is D incorrect? If W is presupposing that only one gene could be responsible for manic depression, wouldn't that automatically disallow any disconfirming evidence?
User avatar
 atomic94
  • Posts: 4
  • Joined: Jul 18, 2021
|
#88865
ashpine17 wrote:But why is D incorrect? If W is presupposing that only one gene could be responsible for manic depression, wouldn't that automatically disallow any disconfirming evidence?
I had the same question -- would love insight on this!
User avatar
 Bob O'Halloran
PowerScore Staff
  • PowerScore Staff
  • Posts: 61
  • Joined: Jul 06, 2021
|
#88931
Hi Atomic94 and Ashpine17,
Thank you for your question.
The question stem here asks us to select the answer that "most accurately expresses Chang’s criticism of Wirth’s argument"
Chang's criticism is that Wirth doesn't account for a combination of genes causing a predisposition. Answer choice (A) basically states this.
Answer choice (D) is not something Chang's criticizes Wirth for directly. You may be able to get there abstractly, but the question is looking for the most accurate.
I hope this helps.
Bob
User avatar
 ashpine17
  • Posts: 321
  • Joined: Apr 06, 2021
|
#99382
but my point still stands: i don't get why D is not an acucrate description of C's criticism of W's argument
User avatar
 ashpine17
  • Posts: 321
  • Joined: Apr 06, 2021
|
#99383
if you're disallowing or not allowing eviddnece you're not taking into consideration other possible evidence?

Get the most out of your LSAT Prep Plus subscription.

Analyze and track your performance with our Testing and Analytics Package.